3
$\begingroup$

Let $3\leq k\leq 8$ be an integer. Suppose $M$ is a complex surface which has a Kahler-Einstein metric and has the same Betti numbers as $\mathbb{C}\mathbb{P}^2\# k\overline{\mathbb{C}\mathbb{P}^2}$, i.e. $$b_0(M)=b_4(M)=1,~~ b_1(M)=b_3(M)=0,~~b_2^+(M)=1,~~ b_2^-(M)=k.$$ Then can we conclude that $M$ is biholomorphic to $\mathbb{C}\mathbb{P}^2\# k\overline{\mathbb{C}\mathbb{P}^2}$?

Note added: The answer of Martin de Borbon has shown that this is not true for $5\leq k\leq 8$. I would like to know if the statement is true for $k=3, 4$.

$\endgroup$
5
  • $\begingroup$ Perhaps you should add simply connected. Otherwise Enriques surfaces satisfy all the above, with k=9 $\endgroup$ Jan 20, 2019 at 11:01
  • 1
    $\begingroup$ @MartindeBorbon Thanks for the comment. I think I understand what you said. And it justifies why I put the condition $3\leq k\leq 8$ in the beginning. $\endgroup$
    – Tong
    Jan 20, 2019 at 11:20
  • $\begingroup$ Right, but I still think you should add the hypothesis that M is simply connected. Otherwise blow ups of fake projective spaces at k points are counter examples $\endgroup$ Jan 20, 2019 at 11:52
  • 1
    $\begingroup$ I am trying to understand what you said....are you saying the blow ups of fake projective spaces have Kahler-Einstein metric? $\endgroup$
    – Tong
    Jan 20, 2019 at 12:01
  • $\begingroup$ my mistake, no these are not, the canonical is not ample $\endgroup$ Jan 20, 2019 at 12:48

1 Answer 1

2
$\begingroup$

There answer is no. The topological manifolds $\mathbb{CP}^2 \sharp k \overline{\mathbb{CP}^2}$ admit smooth structures that support KE with negative scalar curvature for k=5, 6, 7, 8

For k=8, see https://arxiv.org/pdf/dg-ga/9705007.pdf

For k=6, 7, see https://arxiv.org/pdf/0806.1424.pdf

For k=5, see https://msp.org/gt/2009/13-3/gt-v13-n3-p06-p.pdf

$\endgroup$
1
  • $\begingroup$ I see. I wonder if the statement is true for $k=3,4$. I will try to wait and see if there is any result for $k=3,4$. If not, I will accept your answer. Thanks. $\endgroup$
    – Tong
    Jan 20, 2019 at 13:39

Your Answer

By clicking “Post Your Answer”, you agree to our terms of service and acknowledge you have read our privacy policy.

Not the answer you're looking for? Browse other questions tagged or ask your own question.